User Avatar
danielnoahpearlberg295
Joined
Apr 2025
Subscription
Free
User Avatar
danielnoahpearlberg295
Monday, Oct 09 2017

The different prep companies use different labels for the different categories. I like to think of it like this: For about 99% of the games, your task is either to put things in order (basic linear, pure sequencing), put things in groups (grouping), or both put things in order AND put things in groups. For now, don't worry about mapping games and pattern games.

1
User Avatar
danielnoahpearlberg295
Thursday, Aug 11 2016

The reasoning in the stimulus goes like this:

(Premise) Television gave rise to music videos

(Conclusion) Anyone who thinks that music videos are an art form should also agree that television gave rise to an art form [since television gave rise to music videos]

This is a good argument, true by definition.

(C) is actually just one long conditional, rather than an argument: If someone dislikes tomatoes, and if that person agrees that no taste is universal, then that person should agree that some people do like tomatoes.

(E) is not a good argument- if someone prefers fruit to vegetables, and fruit happen to be sweeter than vegetables, this doesn't mean that the person will automatically prefer any X to any Y simply because X is sweeter than Y.

0
User Avatar
danielnoahpearlberg295
Wednesday, Aug 10 2016

I'm not quite sure what you mean by "no need to cover all of them"- many arguments contain more than one flaw, but only one of the answer choices will identify a flaw in the argument. Is that what you're asking?

0
User Avatar
danielnoahpearlberg295
Wednesday, Aug 10 2016

No I basically agree with @lsattrial , I was just pointing out that what the author is failing to address is really central to what's going on in the argument.

"remotely associated to a company"- there are probably lots of ways to be remotely associated, but one example that comes to mind would be if you are casual acquaintances with an employee of the company.

0
User Avatar
danielnoahpearlberg295
Wednesday, Aug 10 2016

The argument is assuming that if the physician's motivations are suspect, then the physician's reasoning in the book is suspect as well. Without this assumption (a very common flawed assumption on the LSAT) the argument falls apart. (D) is just a fancy way of pointing out that the journalist failed to consider that this assumption is flawed- just because the physician might benefit from arguing that the drug isn't dangerous, this in no way means that there is anything wrong with the actual reasoning the physician used to argue that the drug isn't dangerous. So, this answer choice isn't saying that that the journalist "failed to address adequately" any possible consideration that might possibly have a slight effect on the validity of the journalist's argument. It's just saying that he failed to consider the possibility the flawed assumption behind his argument is indeed flawed.

(B) is way too strong in this case. The journalist makes an assumption about a specific individual- the physician- who is more than just "remotely associated" with the company. He gets paid by the company that manufactures the drug, so he will probably benefit if people think that the drug is safe, and he may be harmed a bit (financially) if people think that the drug is not safe.

0
User Avatar
danielnoahpearlberg295
Monday, Aug 01 2016

Haha whoops, sorry about that! Yeah, I think this is a tough question. The conclusion is that the explanation for why we find more impact craters in geologically stable regions must be because those areas have lower rates of destructive geophysical processes. So, why would that have to be the explanation? Well, if we assume that the meteorites causing the impact craters don't actually hit the earth more in some areas than in others, then that means that we should find just as many impact craters in geologically stable regions as we do in geologically unstable regions. So if we actually find more impact craters in the geologically stable regions even though the meteorites weren't hitting those areas any more frequently, it must be because in the geologically stable regions the impact craters are left alone more often than they are in the geologically unstable regions. For example, let's say we have 2 different areas, and each one got hit with 10 meteorites, creating 10 impact craters in each of the areas. However, one of the areas gets lots of earthquakes that mess with the impact craters and so we end up finding less impact craters in the earthquake area than in the non-earthquake area. (Earthquakes are one example of a destructive geophysical process)

So, to reiteratet: If we assume that the geologically stable regions aren't any more likely to be hit with meteorites than are the geologically unstable regions, then the explanation for why we find more impact craters in the geologically unstable regions must be because the geologically unstable regions have lower rates of destructive processes, so the impact craters are being left alone in those regions, in contrast with the geologically stable regions that have higher rates of destructive processes that then mess with the impact craters.

That's why (D) is correct, because it tells us that the geologically stable regions aren't any more likely to be hit with meteorites than are the geologically unstable regions.

1
User Avatar
danielnoahpearlberg295
Sunday, Jul 31 2016

Premise: Lots of highly skilled workers in Eastern Europe left their jobs, moved to the West.

Conclusion: It is likely that the remaining skilled workers in Eastern Europe are in high demand in their home countries.

Well, maybe, but the conclusion doesn't necessarily follow from the premise. Maybe the workers left their jobs because the jobs don't exist anymore? In which case, there's no reason to think that the remaining workers would be in high demand. That would be a good weakener, and it's what answer choice (B) tells us.

(A) would strengthen rather than weaken- if they prefer to hire workers from their home countries, then it should be likely that the remaining workers are in high demand in their home countries.

(C) is irrelevant- what the workers who left now do in the West doesn't affect whether or not the remaining workers are in high demand.

(D) doesn't weaken- if anything, it would strengthen the argument: If they are planning on training new workers, that should imply that the remaining workers that don't need to be trained would be in high demand.

(E) would strengthen rather than weaken- if there are many positions left unfilled, the remaining skilled workers should be in high demand.

Hope that helps :)

0
User Avatar
danielnoahpearlberg295
Friday, Jul 29 2016

Yeah, you have to be careful with flaw questions- many incorrect answer choices describe things that would be flaws if the author of the argument had indeed reasoned in that way, but they are not the correct answer because the author did not in fact reason in that way. In this argument, the author uses the premises that (i) the witness thought the defendant was guilty; and (ii) the jury found the defendant not guilty, to conclude that the not all of the jury members believed the witnesses's testimony. This only makes sense if there is a connection between the witness's testimony and the witnesses's thinking that the defendant was guilty. (E) could have been correct if the argument went like this: (i) the witness gave testimony that many people thought implicated the defendant; (ii) the fury found the defendant not guilty; (conclusion) not all of the jury members believed the witnesses's testimony.

1
User Avatar
danielnoahpearlberg295
Wednesday, Jul 27 2016

Because the conclusion is that Maria trained hard. Think of that as B. We can arrive at that conclusion if we are given the following two premises: (i) If A then B; (ii) A. Then, if those premises are true, the conclusion B must be true as well.

You are suggesting the following premises: (i) If B then A; (ii) A. Even if those premises are true, the conclusion B doesn't follow. That's the mistaken reversal.

0
User Avatar
danielnoahpearlberg295
Wednesday, Jul 27 2016

The legislature's mandate was to hire 500 investigators. The regulator isn't replying by saying that hiring 500 investigators would have been an insufficient response to the scandal (anyways, it would be strange for the regulator to say that- if hiring 500 investigators is insufficient, wouldn't hiring 400 be even worse?). The regulator is saying that they weren't able to hire 500 investigators, due to the low frozen salaries. Which, in turn, challenges the conclusion drawn by the legislator.

1
User Avatar
danielnoahpearlberg295
Thursday, Jul 21 2016

(D) would be the correct answer if, instead of telling us about dexterity, it told us that the uprighters had no more free use of their hands than did the non-uprighters. In fact that would be a pretty good weakener, for the stimulus simply tells us that being upright makes free use of the hands possible- leaving open the possibility that there are plenty of other ways to get free use of the hands. And if being upright isn't necessary for free use of the hands, and the non-uprighters were able to get just as much free use of their hands, then the argument is stupid.

But that's not what happens in (D). Instead, it tells us that the uprighters had no more dexterity than did the non-uprighters. Now, if having no more dexterity implies that they had no more free use of their hands, then (by the reasoning above) (D) would be correct. But having no more dexterity does NOT imply that they had no more free use of their hands. They could have the same level of dexterity, but different levels of "free use of their hands".

So: (D) only weakens the argument if (i)[the uprighters having no more dexterity than the non-uprighters] implies that (ii) [the uprighters have no more free use of hands than the non-uprighters]. But (i) does not imply (ii), so (D) does not weaken the argument.

Hope this helps, sorry for any lack of clarity, this question is definitely starting to make my head spin :)

1
User Avatar
danielnoahpearlberg295
Thursday, Jul 21 2016

Hmm, isn't it possible that the Uprighters and the Non-Uprighters had the exact same amount of dexterity, and the only difference between them (in terms of their hands) was that the Uprighters had free use of the hands and the Non-Uprighters did not?

0
User Avatar
danielnoahpearlberg295
Thursday, Jul 21 2016

I haven't watched the video with J.Y.'s explanation, and I too read (D) as meaning that they were the first who did it- but I think that what's wrong with (D) is that "dexterity" is not the same as "free use of the hands". You can have dexterity without free use, and vice versa.

0
User Avatar
danielnoahpearlberg295
Thursday, Jul 21 2016

Here's one way to look at it: If black water comes into the bay only once every two centuries, the conclusion could still be true- when black water last struck two centuries ago, it did not reach last year's intensity.

2

Confirm action

Are you sure?